Đến nội dung

sinh vien

sinh vien

Đăng ký: 18-04-2015
Offline Đăng nhập: 02-04-2019 - 21:50
-----

#559304 Đa thức + Số học

Gửi bởi sinh vien trong 14-05-2015 - 12:17

Bài toán (Putnam 1977 ) Chứng minh rằng $C_{pa}^{pb}\equiv C_{a}^{b}(mod p)$, với mọi số nghuyên dương a, b và $a\geq b$ , p là số nguyên tố

 Lời giải. Trong lời giải có đề cập đến một số kết quả cơ bản về đa thức trên vành $Z_{p}$

  Theo một kết quả quen thuộc $C_{p}^{i}\equiv 0(modp),i=\overline{1,p-1}$.

Phát biểu lại kết quả trên bằng ngôn ngữ đại số ta được : $(1+x)^{p}=1+x^{p}$.

 Ta thấy

     $\sum_{k=0}^{pa}C_{pa}^{k}x^{k}=(1+x)^{pa}=\left [ (1+x)^{p} \right ]^{a}=(1+x^{p})^{a}=\sum_{j=0}^{a}C_{a}^{j}x^{jp}$

Đồng nhất hệ số của $x^{pb}$ ở hai vế ta thu được

                 $C_{pa}^{pb}\equiv C_{a}^{b}(modp)$

 Lưu ý rằng các tính toán đa thức của ta ở đây đang thực hiện trên $Z_{p}[x]$




#559297 tuyển chọn các bài toán tính định thức

Gửi bởi sinh vien trong 14-05-2015 - 11:14

  Lưu ý với các bạn bài toán này mình đã thay đổi yêu cầu để cho đơn giản bạn có thể tìm thấy nội dung đầy đủ của nó ở Putnam 1984

 Bài toán . Cho n là một số nguyên dương. Gỉa sử a,b,x là các số thực , với $a^{2}\neq b^{2}$, Ma trận vuông $M_{n}$ cấp 2n trong đó phần tử (i,j) được xác định:

$m_{ij}=\left\{\begin{matrix} x & i=j\\ a & i\neq j &i+j =2k \\ b & i\neq j & i+j=2k+1 \end{matrix}\right.$ 

  Tính $detM_{n}$ theo a,b,n

Lời giải. Gọi N là ma trận $M_{n}$ khi x =a . Từ định nghĩa ta dễ dàng nhận thấy N có hạng bằng 2 ( chỉ có hai hàng phân biệt ). Do đó 0 là một giá trị riêng của N với số bội bằng 2n-2.

  Nếu gọi e là ma trận $2n\times 1$ với các phần tử bằng một . 

  Với một chút tính toán ta sẽ thấy Ne=n(a+b)e- điều này có nghĩa là n(a+b) là một giá trị riêng khác của N ứng với vector riêng e

   Ta thấy vết của N bằng 2an $\Rightarrow$ Tồn tại một giá trị riêng thứ ba bằng 2an-n(a+b)=n(a-b)

 Từ đây dễ dàng suy ra rằng

$det(N-\lambda I_{2n})=\lambda ^{2n-2}(\lambda -n(a+b))(\lambda -n(a-b))$

  Để giải quyết trọn vẹn bài toán ta chú ý một điểm quan trọng sau $M_{n}=N-(a-x)I_{2n}$

  Thay $\lambda =a-x$ từ công thức trên ta suy ra

 $detM_{n}=(a-x)^{2n-2}(a-x-n(a-b))(a-x+n(a-b))$.

 




#559288 Đa thức + Số học

Gửi bởi sinh vien trong 14-05-2015 - 09:34

Bài toán (Putnam 2008 ) Cho p là một số nguyên tố . Gỉa sử h(x) là một đa thức hệ số nguyên sao cho $\left \{ h(0),h(1),...,h(p^{2}-1) \right \}$ là một hệ thặng dư đầy đủ modulo $p^{2}$.

 Chứng minh rằng $\left \{ h(0),h(1),...,h(p^{3}-1) \right \}$ lập thành một hệ thặng dư đầy đủ modulo $p^{3}$




#559287 Một ví dụ nhỏ về sử dụng hệ thặng dư

Gửi bởi sinh vien trong 14-05-2015 - 09:29

Bài toán ( Putnam 2007 ) Cho k là một số nguyên dương . Chứng minh rằng tồn tại các đa thức $P_{0}(n),P_{1}(n),...,P_{k-1}(n)$ ( phụ thuộc k ) sao cho với mọi số nghuyên n

              $\left \lfloor \frac{n}{k} \right \rfloor^{k}=P_{0}(n)+P_{1}(n)\left \lfloor \frac{n}{k} \right \rfloor+...+P_{k-1}(n)\left \lfloor \frac{n}{k} \right \rfloor^{k-1}$, trong đó $\left \lfloor x \right \rfloor$ dùng để chỉ số nguyên lớn nhất không vượt quá x.

Lời giải. Mấu chốt của chứng minh nằm ở nhận xét sau :

 Với mọi số nguyên n , Tồn tại  một số $j\in \left \{ 0,1,...,k-1 \right \}$ sao cho $\left \lfloor \frac{n}{k} \right \rfloor=\frac{n-j}{k}$.

  Thật vậy , theo thuật toán chia với mọi số nguyên n , ta đều có thể viết $n=qk+j$ , trong đó $j\in \left \{ 0,...,k-1 \right \}$ nên$q\leq \frac{n}{k}=q+\frac{j}{k}< q+1\Rightarrow \left \lfloor \frac{n}{k} \right \rfloor=q=\frac{n-j}{k}$

 

 Từ nhận xét trên ta thấy 

                $\prod_{j=0}^{k-1}\left ( \left \lfloor \frac{n}{k} \right \rfloor -\frac{n-j}{k}\right )=0$

Đến đây khai triển tích trên ta có ngay điều phải chứng minh




#559215 Đa thức + Số học

Gửi bởi sinh vien trong 13-05-2015 - 20:28

Bài toán . ( Putnam 2011 ) Cho p là một số nguyên tố  lẻ .

Chứng minh rằng có ít nhất $\frac{p+1}{2}$ giá trị $n\in \left \{ 0,1,2... p-1 \right \}$ sao cho $\sum_{k=0}^{p-1}k!n^{k}$ không chia hết cho p.

Lời giải. Đặt $f(x)=\sum_{k=0}^{p-1}k!x^{k}\in F_{p}[x]$ là tập hợp các đa thức bậc p có hệ số thuộc trường $F_{p}$. Ta phân hoạch $F_{p}=Z\bigcup Z^{'}$, 

 trong đó $Z=\left \{ a\in F_{p};f(a)=0 \right \};Z^{'}=\left \{ a\in F_{p} ;f(a)\neq 0\right \}$

 Ta thấy $f(0)=1\Rightarrow 0\in Z^{'}\Rightarrow \left | Z \right |\geqslant 1$.

 Đặt $\left | Z \right |=p-r,\left | Z^{'} \right |=r\geqslant 1$.

 Xét đa thức $s(x)=\prod_{a\in Z^{'}}(x-a)\Rightarrow degs(x)=r$

   Dựa vào các ta định nghĩa thì đa thức $s(x)f(x)$ triệt tiêu ( luôn bằng 0 ) tại mọi điểm của trường $F_{p}$,

Do đó $s(x)f(x)=(x^{p}-x)h(x)$, trong đó h(x) có bậc r-1 và thuộc $F_{p}[x]$ ( Lưu ý đến bậc của hai vế + Định lý Fermat nhỏ ). Trong những tính toán tiếp theo có sử dụng đến đạo hàm hình thức .

$x\frac{d}{dx}(xf(x))=x\frac{d}{dx}\sum_{k=0}^{p-1}k!x^{k+1}=x\sum_{k=0}^{p-2}(k+1)!x^{k}=\sum_{k=1}^{p-1}k!x^{k}=f(x)-1$

$\Rightarrow x^{2}f^{'}(x)+xf(x)=f(x)-1\Rightarrow x^{2}f^{'}(x)+(x-1)f(x)+1=0$

  Lấy đạo hàm hình thức $s(x)f(x)=(x^{p}-x)h(x)$ ta được

      $s^{'}(x)f(x)+s(x)f^{'}(x)=\left ( x^{p}-x \right )h^{'}(x)-h(x)$ ( Chú ý loại bỏ các số hạng triệt tiêu trên trường $F_{p}$ )

 Từ các kết luận trên ta thấy

 $x^{2}s^{'}(x)f(x)=(x^{p}-x)x^{2}h^{'}(x)-x^{2}h(x)-s(x)x^{2}f^{'}(x)$

                          $=(x^{p}-x)x^{2}h^{'}(x)-x^{2}h(x)+s(x)((x-1)f(x)+1)$

                         $=(x^{p}-x)x^{2}h^{'}(x)-x^{2}h(x)+(x-1)(x^{p}-x)h(x)+s(x)$

                         $=\left ( x^{p}-x \right )(x^{2}h^{'}(x)-(x-1)h(x))-x^{2}h(x)+s(x)$

Ta thấy vế trái triệt tiêu trên $Z\bigcup \left \{ 0 \right \}$,  $x^{p}-x$ triệt tiêu trên $F_{p}$ nên đa thức $-x^{2}h(x)+s(x)$ triệt tiêu trên $Z\bigcup\left \{ 0 \right \}$.

Ta thấy

 $r+1=deg(-x^{2}h(x)+s(x))\geqslant \left | Z \right |+1=p-r+1\Rightarrow r\geqslant \frac{p+1}{2}$




#559119 Bộ tài liệu ôn thi olympic môn giải tích

Gửi bởi sinh vien trong 13-05-2015 - 15:19

Bài tập giải tích của Kaczor - Nowak ( 2 cuốn tiếng việt + 1 cuốn tiếng anh )

Tập 1File gửi kèm  problem_analysis1.pdf   2.35MB   4497 Số lần tải

Tập 2 File gửi kèm  BaiTapGiaiTich-Tap2- KaczkorNowak-DoanChi-dich.pdf   2.06MB   4706 Số lần tải

Tập 3 File gửi kèm  Kaczor W.J., Nowak N.T. Problems in mathematical analysis 3. Integration (AMS, 2003)(ISBN 0821832980)(600dpi)(T)(356s)_MCetp_.pdf   12.9MB   5379 Số lần tải

Đề thi một số năm trên diễn đàn có khá nhiều ở đây chỉ nên một số bài mình sở hữu

 1993 - 2005 (  có  Giải tích + Đại số )File gửi kèm  06-10-2014 22.46.37tuyen tap de thi OLP SV toan quoc.pdf   1.27MB   2127 Số lần tải

 2006-2012 ( chỉ Gỉai tích ) File gửi kèm  VNMATH.COM-De Thi Loi Giai Olympic Toan SV Giai Tich 2006 - 2012.rar   759.92K   1983 Số lần tải

2013 ( Giaỉ tích + Đại số , file này tải lâu à nha ) File gửi kèm  7056d1fa-d643-4196-b129-10563fcb9535_kyyeuolympictoan2013.pdf   15.18MB   918 Số lần tải

2014 ( Giaỉ tích + Đại số ) File gửi kèm  OSV2014_Ky_yeu.pdf   2.87MB   1723 Số lần tải

2015  (chỉ Giaỉ tích )File gửi kèm  gt2015.pdf   5.33MB   1315 Số lần tải

 Cơ bản ôn nhiêu chắc cũng chủ đậu rồi




#558175 tuyển chọn các bài toán tính định thức

Gửi bởi sinh vien trong 07-05-2015 - 12:35

Bài toán:(Putnam 2009 )Cho n > 3, n nguên dương. Tính định thức:

 $D_{n}=\begin{vmatrix} cos1 &cos2 &... &cosn \\ cos(n+1) & cos(n+2) &... &cos2n \\... &... &... &... \\cos(n^{2}-n+1) & cos(n^{2}-n+2) &... &cosn^{2} \end{vmatrix}$

Lời giải.  Lấy cột thứ 3 + cột thứ 1$\rightarrow$ cột thứ 1 và sử dụng biến đổi lượng giác , ta được :

$D_{n}=\begin{vmatrix} 2cos2cos1 &cos2 &... & ... &... cos(n) \\2cos(n+2)cos1 &cos(n+2) &... & ... &cos2n \\... & ... & & ... &... \\... &... & ... & ... &... \\2cos(n^{2}-n+2)cos1 & cos(n^{2}-n+2) &... &... & cosn^{2} \end{vmatrix}$

   Lúc này cột thứ 1= 2cos1 cột thứ 2 $\Rightarrow$ det$D_{n}=0$.

Bài toán . Xét các số phức$z_{1},z_{2},...,z_{2n}$ thỏa mãn điều kiện  $\left | z_{1} \right |=\left | z_{2} \right |=...=\left | z_{n+3} \right |$ và $argz_{1}\geqslant argz_{2}\geqslant ...\geq argz_{n+3}$.

  Tính định thức của ma trận B ,trong đó  $b_{ij}=\left | z_{i}-z_{j+n} \right |,i,j\in \left \{ 1,2,...n \right \}$.

 Lời giải . Ta có nhận xét sau bài toán đề cập đến modun và argument của một số phức nên ta sẽ sử dụng dạng lượng giác của số phức trong các tính toán. 

    Với hai số phức $z=r(cosx+isinx);\omega =r(cosy+isiny)\Rightarrow \left | z-\omega \right |=2r\left | sin\left ( \frac{x-y}{2} \right ) \right |$. 

  Kết hợp với điều kiện ở đầu bài, ta thấy B có dạng như sau:

$B=(2r)^{n}\begin{vmatrix} sin(x_{1}-x_{n+1}) &sin(x_{1}-x_{n+2}) & sin(x_{1}-x_{n+3}) & ...\\... &... & ... &... \\ sin(x_{n}-x_{n+1}) & sin\left ( x_{n}-x_{n+2} \right ) & sin(x_{n}-x_{n+3}) & ... \end{vmatrix}$

 trong đó $x_{i}$ kí hiệu tương ứng argument của $z_{i}$ và r=$\left | z_{1} \right |=\left | z_{2} \right |=...=\left | z_{2n} \right |$

 Nếu ta tách det B theo các cột thì dễ dàng nhận thấy det B=0 

 

 




#558174 tuyển chọn các bài toán tính định thức

Gửi bởi sinh vien trong 07-05-2015 - 12:35

Bài toán:(Putnam 2009 )Cho n > 3, n nguên dương. Tính định thức:

 $D_{n}=\begin{vmatrix} cos1 &cos2 &... &cosn \\ cos(n+1) & cos(n+2) &... &cos2n \\... &... &... &... \\cos(n^{2}-n+1) & cos(n^{2}-n+2) &... &cosn^{2} \end{vmatrix}$

Lời giải.  Lấy cột thứ 3 + cột thứ 1$\rightarrow$ cột thứ 1 và sử dụng biến đổi lượng giác , ta được :

$D_{n}=\begin{vmatrix} 2cos2cos1 &cos2 &... & ... &... cos(n) \\2cos(n+2)cos1 &cos(n+2) &... & ... &cos2n \\... & ... & & ... &... \\... &... & ... & ... &... \\2cos(n^{2}-n+2)cos1 & cos(n^{2}-n+2) &... &... & cosn^{2} \end{vmatrix}$

   Lúc này cột thứ 1= 2cos1 cột thứ 2 $\Rightarrow$ det$D_{n}=0$.

Bài toán . Xét các số phức$z_{1},z_{2},...,z_{2n}$ thỏa mãn điều kiện  $\left | z_{1} \right |=\left | z_{2} \right |=...=\left | z_{n+3} \right |$ và $argz_{1}\geqslant argz_{2}\geqslant ...\geq argz_{n+3}$.

  Tính định thức của ma trận B ,trong đó  $b_{ij}=\left | z_{i}-z_{j+n} \right |,i,j\in \left \{ 1,2,...n \right \}$.

 Lời giải . Ta có nhận xét sau bài toán đề cập đến modun và argument của một số phức nên ta sẽ sử dụng dạng lượng giác của số phức trong các tính toán. 

    Với hai số phức $z=r(cosx+isinx);\omega =r(cosy+isiny)\Rightarrow \left | z-\omega \right |=2r\left | sin\left ( \frac{x-y}{2} \right ) \right |$. 

  Kết hợp với điều kiện ở đầu bài, ta thấy B có dạng như sau:

$B=(2r)^{n}\begin{vmatrix} sin(x_{1}-x_{n+1}) &sin(x_{1}-x_{n+2}) & sin(x_{1}-x_{n+3}) & ...\\... &... & ... &... \\ sin(x_{n}-x_{n+1}) & sin\left ( x_{n}-x_{n+2} \right ) & sin(x_{n}-x_{n+3}) & ... \end{vmatrix}$

 trong đó $x_{i}$ kí hiệu tương ứng argument của $z_{i}$ và r=$\left | z_{1} \right |=\left | z_{2} \right |=...=\left | z_{2n} \right |$

 Nếu ta tách det B theo các cột thì dễ dàng nhận thấy det B=0 

 

 




#558170 Hệ phương trình vi phân loại khó

Gửi bởi sinh vien trong 07-05-2015 - 11:55

Bài toán ( Putnam 2009) . Gỉa sử $f,g,h$ là các hàm khả vi trên một khoảng mở chứa điểm 0 Và thỏa mãn :

$f^{'}=2f^{2}gh+\frac{1}{gh};g^{'}=fg^{2}h+\frac{4}{fh};h^{'}=3fgh^{2}+\frac{1}{fg}$ 

 và $f(0)=1;g(0)=1;h(0)=1$.

 Tìm một biểu thức cho hàm f(x) , giả thiết f(x) trong triệt tiêu trên khoảng mở chứa 0.

Lời giải. Nhân đẳng thức thứ nhất cho gh , thứ 2 cho fh, và đẳng thức cuối cho fg ta được

    $f^{'}gh=2\left ( fgh \right )^{2}+1;fg^{'}h=(fgh)^{2}+4;fgh^{'}=3(fgh)^{2}+1$.

 Cộng các đẳng thức này ta được

    $f^{'}gh+fg^{'}h+fgh^{'}=6(fgh)^{2}+6\Rightarrow (fgh)^{'}=6(fgh)^{2}+6$.

 Bằng cách đặt k(x)=f(x)g(x)h(x); ta được $k^{'}=6k^{2}+6\Rightarrow \frac{\mathrm{d} k}{\mathrm{d} x}=6k^{2}+6\Rightarrow \frac{dk}{6k^{2}+6}=dx$

 Kết hợp với điều kiện k(0)=1 ta thấy: $k(x)=tan\left ( 6x+\frac{\pi }{4} \right ).$

   Lại thấy:$\frac{f^{'}}{f}=2tan\left ( 6x+\frac{\pi }{4} \right )+cot\left ( 6x+\frac{\pi }{4} \right )$

$\Rightarrow lnf(x)=\frac{-2lncos\left ( 6x+\frac{\pi }{4} \right )+lnsin\left ( 6x+\frac{\pi }{4} \right )}{6}$

$\Rightarrow f(x)=e^{c}\left ( \frac{sin\left ( 6x+\frac{\pi }{4} \right )}{cos^{2}\left ( 6x+\frac{\pi }{4} \right )} \right )^{\frac{1}{6}}$. Từ giả thiết f(0)=1 ta tính được

   $f(x)=\frac{1}{2^{12}}\left ( \frac{sin\left ( 6x+\frac{\pi }{4} \right )}{cos^{2}\left ( 6x+\frac{\pi }{4} \right )} \right )^{\frac{1}{6}}$.

  Lưu ý là ta cũng có thể tính được g(x), h(x) bằng các tương tự.




#557905 Ứng dụng giải tích trong các bài toán số học

Gửi bởi sinh vien trong 05-05-2015 - 08:46

Bài toán.  Cho n là một số nguyên lớn hơn bằng 2.

Chứng minh rằng

 $\sum_{p\leq n}\frac{1}{p}> lnlnn-1$, trong đó ta quy ước p chỉ các số nguyên tố nên tổng ở đây lấy trên tập các số nguyên tố nhỏ hơn bằng n.

Lời giải. Gọi A(n) là tập các số nguyên dương sao cho các ước số nguyên tố nhỏ hơn hoặc bằng n.

Khi đó

 $\prod_{p\leq n}\left ( 1+\frac{1}{p}+\frac{1}{p^{2}}+...+\frac{1}{p^{k}}+.... \right )=\sum_{m\in A(n)}\frac{1}{m}$ (1)

Ta thấy rằng $\sum_{m\in A(n)}\frac{1}{m}\geq \sum_{m=1}^{n}\frac{1}{m}$ (định nghĩa của tập A(n) )

mà ta lại có bất đẳng thức sau ( chứng minh không quá khó khăn )   $\sum_{m=1}^{n}\frac{1}{m}> lnn$  . Lưu ý rằng ta cũng có đẳng thức sau :$1+\frac{1}{p}+\frac{1}{p^{2}}+...+\frac{1}{p^{k}}+...=\left (1-\frac{1}{p} \right )^{-1}$ nên ta có thể viết lại  (1) như sau$\prod_{p\leq n}\left ( 1-\frac{1}{p} \right )^{-1}> lnn$.

 Để thực hiện tiếp phép chứng minh ta cần đến một bất đẳng thức không tầm thường :

$e^{t+t^{2}}\geq \left ( 1-t \right )^{-1}, 0\leq t\leq \frac{1}{2}$ có thể được chứng minh dễ dàng bằng  phương pháp lấy đạo hàm :

  Do đó :

    $\prod_{p\leq n}e^{\frac{1}{p}+\frac{1}{p^{2}}}\geq \prod_{p\leq n}\left ( 1-\frac{1}{p} \right )^{-1}> lnn$.(2)

 Từ (2) không quá khó để nhận ra rằng $\sum_{p\leq n}\frac{1}{p}+\sum_{p\leq n}\frac{1}{p^{2}}>lnlnn\Rightarrow \sum_{p\leq n}\frac{1}{p}> lnlnn-\sum_{p\leq n}\frac{1}{p^{2}}$

 mà $\sum_{p\leq n}\frac{1}{p^{2}}\leq \sum_{k=2}^{\infty }\frac{1}{k^{2}}=\frac{\pi ^{2}}{6}-1< 1$ (3)

  Từ (2) và (3) ta đi đến :$\sum_{p\leq n}\frac{1}{p}> lnlnn-1$ (đpcm)

         




#555864 Chứng minh tích phân bằng tổ hợp

Gửi bởi sinh vien trong 23-04-2015 - 19:37

Bài toán.(Putnam ? ) Hỏi với giá trị của n như thế nào thì tích phân

    $I=\int_{0}^{2\pi }cosxcos2x...cosnxdx$ có giá trị khác không ? , 

Lời giải .

Áp dụng định lí Morvie : $e^{ix}=cosx+isinx$ ta dễ dàng suy ra $cosx=\frac{1}{2}(e^{ix}+e^{-ix})$ thay vào biểu thức của $I$ ta thấy

$I=\frac{1}{2^{n}}\int_{0}^{2\pi }(e^{ix}+e^{-ix})(e^{2ix}+e^{-2ix})...(e^{nix}+e^{-nix})dx=\frac{1}{2^{n}}\int_{0}^{2\pi }\sum_{\varepsilon _{1},\varepsilon _{2}...\varepsilon _{n}=\pm 1}e^{(\varepsilon _{1}+2\varepsilon _{2}+...n\varepsilon _{n})ix}dx$=

  $\frac{1}{2^{n}}\sum_{\varepsilon _{1},...\varepsilon _{n}=\pm 1}\int_{0}^{2\pi }e^{(\epsilon _{1}+2\varepsilon _{2}+...+n\varepsilon _{n})ix}dx$

  Ta có :   $\int_{0}^{2\pi }e^{mix}dx=2\pi$ nếu $m=0$ và bằng 0 trong trường hợp $m\neq 0$. 

Do đó: $I=\frac{\pi}{2^{n-1}}S(n)$

trong đó $S(n)$ là số cách chọn các dấu +, -  sao cho $\pm 1\pm 2\pm...\pm n=0$.(Do n=0 hiển nhiên làm cho tích phân ban đầu khác không nên ở đây ta chỉ xét các giá trị $n\geq 1$ )

    Bài toán của ta trở thành tìm n sao cho $S(n)\neq 0$.    

Ta xét các trường hợp của n

 Nếu $n=4k$ thì từ đẳng thức : $(1-2-3+4)+(5-6-7+8)+...+((4k-3)-(4k-2)-(4k-1)+4k)=0$ ta suy ra $S(n)\neq 0$  

Nếu n=4k+3  thì từ đẳng thức: $(1+2-3)+(4-5-6+7)+...+((4k)-(4k+1)-(4k+2)+(4k+3))=0$ ta cũng suy ra $S(n)\neq 0$

Nếu n=4k+1 hoặc 4k+2:

   $\varepsilon _{1}+2\varepsilon _{2}+...+n\varepsilon _{n}\equiv 1+2+...+n=\frac{n(n+1)}{2}\not\equiv 0 mod2$  , trong đó $\varepsilon _{1},...\varepsilon _{n}=\pm 1$ nên $S(n)=0$

  Từ khảo sát trên ta dễ thấy giá trị cần tìm là n=4k , n=4k+3 , trong đó k là số tự nhiên nào đó

 

  




#555380 tuyển chọn các bài toán tính định thức

Gửi bởi sinh vien trong 21-04-2015 - 08:53

Sau đây là một số bài toán tính định thức theo các hướng độc đáo đã được sau trên các cuộc thi danh tiếng  (đề mục này sẽ được bổ sung dần dần )

   

Bài toán 1. (PUTNAM 2014)  Cho A là ma trận $n\times n$ trong đó phần tử ở hàng thứ i cột thứ j được cho bởi $\frac{1}{min(i,j)}$ với $1\leqslant i,j\leqslant n$ . Tính định thức của ma trận A

Lời giải. Từ điều kiện ở đầu ta thấy 

           A=$\begin{bmatrix} 1 & 1 &1 ...& 1& \\ 1& \frac{1}{2} &\frac{1}{2} ...& \frac{1}{2}& \\ 1 & \frac{1}{2}&\frac{1}{3} ...&\frac{1}{3} & \\1 &\frac{1}{2}&\frac{1}{3}...& \frac{1}{4}\\ ... & ... & ... &... \\ 1 & \frac{1}{2}&\frac{1}{3} ... & \frac{1}{n} & \end{bmatrix}$

   Để giải bài toán này ta sẽ áp dụng khai triển Laplace  cho hàng thứ n . Chú ý khi đó ma trận con thu được khi xóa hàng thứ n có dạng 

         \begin{bmatrix} 1 & 1& 1 & ... &1& 1\\1 & \frac{1}{2}&\frac{1}{2} & ...& \frac{1}{2} & \frac{1}{2}\\ ... & ... &... & ...&... & ...\\1 & \frac{1}{2}&\frac{1}{3} &..& \frac{1}{n-1} &\frac{1}{n-1} \end{bmatrix}$

Khi đó cột thứ n và cột thứ n-1 trùng nhau nên ma trận con chứa cả hai cột n và n-1 có định thức bằng 0 . Từ đó ta thấy

        $\begin{vmatrix} 1& 1 & 1 & ... &1 \\1 & \frac{1}{2} & \frac{1}{2} &... & \frac{1}{2}\\ 1& \frac{1}{2}&\frac{1}{2} &... &\frac{1}{2} \\ ...& ... &... & ... &... \\ 1& \frac{1}{2} &\frac{1}{3} & ... &\frac{1}{n} \end{vmatrix}=-\frac{1}{n-1}\begin{vmatrix} 1 & 1 & ... &1 \\ 1 & \frac{1}{2} &... &\frac{1}{2} \\... & ...& ... & ...\\ 1 & \frac{1}{2}&... &\frac{1}{n-1} \end{vmatrix}+\frac{1}{n}\begin{vmatrix} 1 & 1 & ... & 1\\1 & \frac{1}{2} & ... & \frac{1}{2}\\ ... &\... & ... &... \\ 1 & \frac{1}{2} &... &\frac{1}{n-1} \end{vmatrix}$

    Nếu đặt $D_{n}=det(A)$ trong đó A có cấp n thì $D_{n}=\left ( \frac{1}{n} -\frac{1}{n-1}\right ) D_{n}=\frac{-1}{n(n-1)}D_{n-1}$

    Sử dụng hệ thức truy hồi này và chú ý $D_{1}=1$ ta được

$D_{n}=\frac{-1}{n(n-1)}\frac{-1}{(n-1)(n-2)}...\frac{-1}{3.2}\frac{-1}{2.1}=\frac{(-1)^{n+1}}{n!(n-1)!}$

   Bài toán 2 (ĐHBK -2013) .Cho $x_{i},y_{i},1\leq i\leq n$ là các số phức với $x_{i}y_{j}$$\neq 1$ với mọi cặp $x_{i},y_{j}$.

  Tính định thức của ma trận $M=(m_{ij})_{n\times n}$ tróng đó $m_{ij}=\frac{1}{1-x_{i}y_{j}}$

lời giải. Để cho thuận tiện ta quy ước $D_{y_{1}y_{2}...y_{n}}^{x_{1}x_{2}...x_{n}}=det(M)$. Ta thấy

 n=2 : $D_{y_{1}y_{2}}^{x_{1}x_{2}}=\begin{vmatrix} \frac{1}{1-x_{1}y_{1}} &\frac{1}{1-x_{1}y_{2}} \\ \frac{1}{1-x_{2}y_{1}} &\frac{1}{1-x_{2}y_{2}} \end{vmatrix}=\frac{(x_{1}-x_{2})(y_{1}-y_{2})}{(1-x_{1}y_{1})(1-x_{1}y_{2})(1-x_{2}y_{1})(1-x_{2}y_{2})}$  ( tính toán tương đối đơn giản nên mình không nêu ra cụ thể )

Ta có thể tính trực tiếp thêm một số giá trị của n . Dự đoán :

$D_{y_{1}...y_{n}}^{x_{1}...x_{n}}=\frac{\prod_{1\leqslant i< j\leqslant n}(x_{i}-x_{j})(y_{i}-y_{j})}{\prod_{1\leq i,j\leq n}(1-x_{i}y_{j})}$. Ta sẽ chứng minh quy nạp công thức này.

   Áp dụng khai triển Laplace cho cột thứ nhất ta được

$D_{y_{1}..y_{n+1}}^{x_{1}...y_{n+1}}=\sum_{i=1}^{n+1}(-1)^{i+1}\frac{1}{1-x_{i}y_{1}}D_{y_{2}...y_{n+1}}^{x_{1}...x_{i-1}x_{i+1}...x_{n+1}}$

$=\sum_{i=1}^{n+1}(-1)^{i-1}\frac{1}{1-x_{i}y_{1}}\frac{\prod_{1\leq k< l\leq n+1,k,l\neq i}(x_{k}-x_{l})\prod_{2\leq k< l\leq n+1}(y_{k}-y_{l})}{\prod_{1\leq k\leqslant n+1,k\neq i,2\leq l\leqslant n+1}(1-x_{k}y _{l})}$

$\sum_{i=1}^{n+1}(-1)^{i-1}\prod_{k=1,k\neq i}^{n+1}(1-x_{k}y_{1})\frac{\prod_{1\leqslant k< l\leq n+1}(x_{k}-x_{l})\prod_{k=2}^{n+1}(1-x_{i}y_{k})\prod_{2\leq k< l\leq n+1}(y_{k}-y_{l})}{\prod_{k=1}^{i-1}(x_{k}-x_{i})\prod_{k=i+1}^{n+1}(x_{i}-x_{k})\prod_{1\leqslant k,l\leqslant n+1}(1-x_{k}y_{l})}$

$=\left ( \sum_{i=1}^{n+1}\prod_{k=2}^{n+1}(1-x_{i}y_{k})\prod_{k=1,k\neq i}^{n+1} \frac{1-x_{k}y_{1}}{x_{i}-x_{k}}\right )\frac{\prod_{1\leq k< l\leq n+1}(x_{k}-x_{l})\prod_{2\leq k< l\leq n+1}(y_{k}-y_{l})}{\prod_{1\leqslant k,l\leq n+1}(1-x_{k}y_{l})}$

 Đặt $P(x)=\prod_{k=2}^{n+1}(1-xy_{k})$ thì $P(x)$ là một đa thức bậc n và $P(x_{i})=\prod_{k=2}^{n+1}(1-x_{i}y_{k}),i=1,2,..n+1$ . Áp dụng công thức Lagrange ta được:

  $P(x)=\sum_{i=1}^{n+1}\prod_{k=2}^{n+1}(1-x_{i}y_{k})\prod_{k=1,k\neq i}^{n+1}\frac{x-x_{k}}{x_{i}-x_{k}}$

 Thay $x=\frac{1}{y_{1}}$ , nhân hai vế cho $y_{1}^{n}$ ta được

      $\prod_{k=2}^{n+1}(y_{1}-y_{k})=\sum_{i=1}^{n+1}\prod_{k=2}^{n+1}(1-x_{i}y_{k})\prod_{k=1,k\neq i}^{n+1}\frac{1-x_{k}y_{1}}{x_{i}-x_{k}}$.

  Từ đây ta dễ dàng thu được kết quả mong muốn.




#555192 Tài liệu đề thi thạc sĩ của Rumani

Gửi bởi sinh vien trong 20-04-2015 - 09:02

 Sau hai bộ '' full' nam và ' baltic way ' thì đây là bộ tài liệu khó và còn bị ít mong các bạn thông cảm

   lần 2File gửi kèm  solutions2009.pdf   99.41K   169 Số lần tải

  lân 4 ngày 1File gửi kèm  Sols2011D1.pdf   62.76K   109 Số lần tải ngày 2 File gửi kèm  Sols2011D2.pdf   99.56K   100 Số lần tải

lần 5 ngày 1 File gửi kèm  Solutions2012-1.pdf   60.69K   97 Số lần tải ngày 2File gửi kèm  Solutions2012-2.pdf   67.87K   99 Số lần tải

lần 6 ngày 1File gửi kèm  Solutions2013-1.pdf   188.4K   201 Số lần tải ngày 2 File gửi kèm  Solutions2013-2.pdf   183.82K   115 Số lần tải

lân 7 ngày 1File gửi kèm  Solutions_RMM2015-1.pdf   166.11K   93 Số lần tải ngày 2 File gửi kèm  Solutions_RMM2015-2.pdf   173.35K   95 Số lần tải

  Nếu có bạn nào có các lần thi bị khuyết xin hãy chia sẻ ở đây nhé !




#555191 Tuyển tập đề thi Baltic Way

Gửi bởi sinh vien trong 20-04-2015 - 08:44

Sau bộ tài liệu putnam đây là một bộ tài liệu dành cho phổ thông tốt :

1990 -File gửi kèm  bw90sol.pdf   61.88K   133 Số lần tải

1991File gửi kèm  bw91sol.pdf   69.69K   152 Số lần tải

1992File gửi kèm  bw92sol.pdf   73.12K   135 Số lần tải

1993File gửi kèm  bw93sol.pdf   80.08K   153 Số lần tải

1994File gửi kèm  bw94sol.pdf   87.38K   125 Số lần tải

1995File gửi kèm  bw95sol.pdf   73.61K   145 Số lần tải

1996File gửi kèm  bw96sol.pdf   81.36K   108 Số lần tải

1997File gửi kèm  bw97sol.pdf   123.31K   102 Số lần tải

1998File gửi kèm  bw98sol.pdf   121.46K   108 Số lần tải

1999File gửi kèm  bw99sol.pdf   124.71K   108 Số lần tải

2000File gửi kèm  bw00sol.pdf   102.75K   104 Số lần tải

2001File gửi kèm  bw01sol.pdf   95.83K   114 Số lần tải

2002File gửi kèm  bw02sol.pdf   121.4K   120 Số lần tải

2003File gửi kèm  bw03sol.pdf   157.74K   149 Số lần tải

2004File gửi kèm  bw04sol.pdf   140.7K   109 Số lần tải

2005File gửi kèm  bw05sol.pdf   144.51K   164 Số lần tải

2006File gửi kèm  bw06sol.pdf   145.27K   100 Số lần tải

2007File gửi kèm  bw07sol.pdf   207.33K   104 Số lần tải

2008File gửi kèm  bw08sol.pdf   159.89K   105 Số lần tải

2009File gửi kèm  bw09pb.pdf   49.85K   116 Số lần tải (năm này không thấy có lời giải cụ thể các bạn có thể tham khảo trên trang '' art problem solving'' các lời giải của từng bài )

2010File gửi kèm  bw10sol.pdf   224.59K   155 Số lần tải

2011File gửi kèm  bw11sol.pdf   352.37K   136 Số lần tải

2012File gửi kèm  bw12sol.pdf   204.93K   255 Số lần tải

2013File gửi kèm  PROBSOLS.pdf   1.29MB   237 Số lần tải

     Mong rằng với số lượng tài liệu ít ỏi này sẽ giúp ít cho nhiều bạn




#555030 Tài liệu gồm 4 tập sách của Ramanujan

Gửi bởi sinh vien trong 19-04-2015 - 12:05

Đây là một bộ tài liệu rất hữu ích cho những ai đam mê giải tích

Quyển 1 File gửi kèm  Ramanujan's Notebooks I.pdf   12.41MB   971 Số lần tải

Quyển 2 File gửi kèm  Ramanujan's Notebooks II.pdf   13.64MB   406 Số lần tải

Quyển 3 File gửi kèm  Ramanujan's Notebooks III.pdf   20.03MB   405 Số lần tải 

Quyển 4 File gửi kèm  Ramanujan's Notebooks IV.pdf   10.63MB   335 Số lần tải

 còn đây là  cuốn sách lý thuyết số - giải tích tổng hợp cô động các thành tựu của ramanujan

 File gửi kèm  Bruce C. Berndt - Number Theory in the Spirit of Ramanujan - AMS - 201p.pdf   14.02MB   501 Số lần tải